Difference between revisions of "2021 AMC 10B Problems/Problem 1"

(Solution)
(Removed all unnecessary contents. I merged all contents on the corresponding AMC 12 page.)
(Tag: Replaced)
 
(15 intermediate revisions by 10 users not shown)
Line 1: Line 1:
==Problem==
+
#REDIRECT [[2021 AMC 12B Problems/Problem 1|Solution]]
How many integer values of <math>x</math> satisfy <math>|x|<3\pi</math>?
 
 
 
<math>\textbf{(A)} ~9 \qquad\textbf{(B)} ~10 \qquad\textbf{(C)} ~18 \qquad\textbf{(D)} ~19 \qquad\textbf{(E)} ~20</math>
 
 
 
==Solution==
 
Since <math>3\pi</math> is about <math>9.42</math>, we multiply 9 by 2 and add 1 to get <math> \boxed{\textbf{(D)}\ ~19} </math>~smarty101
 
 
 
==Solution 2==
 
<math>3\pi \approx 9.4.</math> There are two cases here.
 
 
 
When <math>x>0, |x|>0,</math> and <math>x = |x|.</math> So then <math>x<9.4</math>
 
 
 
When <math>x<0, |x|>0,</math> and <math>x = -|x|.</math> So then <math>-x<9.4</math>. Dividing by <math>-1</math> and flipping the sign, we get <math>x>-9.4.</math>
 
 
 
From case 1 and 2, we need <math>-9.4 < x < 9.4</math>. Since <math>x</math> is an integer, we must have <math>x</math> between <math>-9</math> and <math>9</math>. There are a total of <cmath>9-(-9) + 1 = \boxed{\textbf{(D)}\ ~19} \text{ integers}.</cmath>
 
 
 
-PureSwag
 

Latest revision as of 05:16, 4 March 2021